13
$\begingroup$

Suppose $X$ is a smooth projective variety over a field $k$, with ample canonical bundle. If $\operatorname{char}(k)=0$ or $\operatorname{char}(k)>\dim(X)$ and $X$ lifts to $W_2(k)$ (thanks Olivier Benoist!), it's not hard to see that $X$ has finite automorphism group.

Proof. Let $X\hookrightarrow \mathbb{P}\Gamma(X, \omega_X^{\otimes n})^\vee$ be a closed embedding. Then the induced map $\operatorname{Aut}(X)\hookrightarrow PGL(\Gamma(X, \omega_X^{\otimes n})^\vee)$ is a closed embedding, so $\operatorname{Aut}(X)$ is finite type. But the tangent space to $\operatorname{Aut}(X)$ at the identity is $\Gamma(X, T_X)$, which is trivial by Kodaira vanishing. So $\operatorname{Aut}(X)$ is a $0$-dimensional finite type group scheme, hence finite. $\blacksquare$

(An essentially identical argument avoiding the representability of $\operatorname{Aut}(X)$ may be found here.)

This proof breaks down for $\operatorname{char}(k)$ small: $\Gamma(X, T_X)$ need not be trivial. But I don't know of an example of a canonically polarized variety with infinite (thus necessarily positive-dimensional) automorphism group in small characteristic.

Is the automorphism group of a smooth variety with ample canonical bundle always finite, even if $\operatorname{char}(k)$ is small?

$\endgroup$
10
  • 1
    $\begingroup$ How do you know that the map ${\rm{Aut}}_{X/k} \rightarrow {\rm{PGL}}(\Gamma(X,\omega^{\otimes n})^{\vee})$ has closed image (e.g., why is it finite type?)? It is certainly not sufficient that it has trivial kernel. One can make plenty of maps from a locally finite type $k$-group to a finite type $k$-group with trivial kernel. $\endgroup$
    – user76758
    Dec 21, 2013 at 1:19
  • $\begingroup$ The proof in your link is also flawed; why should $v$ there have anything to do with infinitesimal automorphisms of $X$ if it does not arise from the tangent space to the automorphism scheme (but just that of the closure of its image)? $\endgroup$
    – user76758
    Dec 21, 2013 at 1:23
  • 2
    $\begingroup$ @user76758 : no it is OK : consider the closed subgroup scheme $G\subset PGL_{N+1}$ consisting of automorphisms preserving $X$. There are natural morphisms of group schemes in both directions : $G\to Aut(X)$ by restricting the action to $X$, and $Aut(X)\to G$ using the induced action on pluricanonical forms. Using the fact that $X\to\mathbb{P}^N$ is an embedding, you can see that these two maps are inverse to each other. $\endgroup$ Dec 21, 2013 at 1:32
  • $\begingroup$ @DanielLitt : I am worried about your statement that you will be able to apply Kodaira vanishing if $p>dim(X)$. Are you sure you're not mistaken ? $\endgroup$ Dec 21, 2013 at 1:34
  • 4
    $\begingroup$ @DanielLitt : to apply Deligne-Illusie, you also need to know that $X$ lifts to the second Witt vectors of $k$. $\endgroup$ Dec 21, 2013 at 2:05

1 Answer 1

17
$\begingroup$

A much more general result is proven in Martin-Deschamps, Lewin-Menegaux : Applications rationnelles séparables dominantes sur une variété de type général. Théorème 2 : if $X$ and $Y$ are smooth and proper, and if $Y$ is of general type, the set of dominant separable rational maps from $X$ to $Y$ is finite ! What you want follows immediately: it is their Corollaire 4, and they attribute it to Matsumura.

$\endgroup$
1
  • $\begingroup$ Perfect! I'd seen Kobayashi's analytic argument for this over $\mathbb{C}$, but hadn't been able to find an algebraic argument. $\endgroup$ Dec 21, 2013 at 2:03

Your Answer

By clicking “Post Your Answer”, you agree to our terms of service and acknowledge you have read our privacy policy.

Not the answer you're looking for? Browse other questions tagged or ask your own question.